
Today we will use induction to solve some more problems.
Problem 4. Prove that 1 \times 2 \times 3 + 2 \times 3 \times 4 + \dots + n (n+1)(n+2) = \frac{1}{4} n(n+1)(n+2)(n+3).
Solution. We will prove by induction the following formula, 1 \times 2 \times 3 + 2 \times 3 \times 4 + \dots + n (n+1)(n+2) = \frac{1}{4} n(n+1)(n+2)(n+3).
So the above formula is correct for the case n=1.
Assume that for any 1 \leq n \leq k, the formula is correct. We will prove that the formula is also correct for the case n=k+1, i.e., we will prove that 1 \times 2 \times 3 + \dots + k (k+1)(k+2) + (k+1)(k+2)(k+3) = \frac{1}{4} (k+1)(k+2)(k+3)(k+4).
Indeed, by induction assumption, the formula is correct for the case n=k, so 1 \times 2 \times 3 + 2 \times 3 \times 4 + \dots + k (k+1)(k+2) = \frac{1}{4} k(k+1)(k+2)(k+3).
Hence, 1 \times 2 \times 3 + \dots + k (k+1)(k+2) + (k+1)(k+2)(k+3) = \frac{1}{4} k(k+1)(k+2)(k+3) + (k+1)(k+2)(k+3) = \frac{1}{4} (k+1)(k+2)(k+3)(k+4).
We have proved that the formula is correct for the case n=k+1.
By the mathematical induction principle, the formula must be correct for any n \geq 1. \blacksquare
Problem 5. Prove that 49 ~\mid~ 8^n + 42 n - 1.
Solution. We will prove by induction the following statement 8^n + 42 n - 1 = 0 \pmod{49}
For n=0, we have 8^0 + 42 \times 0 - 1 = 0
So the above statement is correct for the case n=0.
Assume that the statement is correct for any 0 \leq n \leq k. We will prove that the statement is also correct for the case n=k+1, i.e., we will prove that 8^{k+1} + 42 (k+1) - 1 = 8^{k+1} + 42 k + 41 = 0 \pmod{49}.
Indeed, by the induction assumption, the statement is correct for the case n=k, so we have 8^k + 42 k - 1 = 0 \pmod{49} .
Thus, 8(8^k + 42 k - 1) = 8^{k+1} + 336 k - 8 = 0 \pmod{49} .
Therefore, 8^{k+1} + 42 k + 41 = (8^{k+1} + 336 k - 8) - 49(6k - 1) = 0 \pmod{49} .
We have proved that the statement is correct for the case n=k+1.
By the mathematical induction principle, the statement must be correct for any natural number n. \blacksquare
In the above problems, we can see that in the induction step, in order to prove that P(k+1) is correct, we only need to use the assumption that P(k) is correct. We did not need to use the assumption that P(0), P(1), ..., P(k-1) are correct.
In the next problem, we will see that in order to prove that P(k+1) is correct, we need to use both the assumption that P(k-1) is correct and the assumption that P(k) is correct.
Problem 6. The Fibonacci sequence is determined as follows: F_0 = 0, F_1 = 1, F_{n+1} = F_n + F_{n-1}. Thus,
F_0 = 0, F_1 = 1, F_2 = 1, F_3 = 2, F_4 = 3, F_5 = 5, F_6 = 8, \dots
Prove that the formula for Fibonacci sequence is
F_n = \frac{1}{\sqrt{5}} \left[ \left( \frac{1 + \sqrt{5}}{2} \right)^n - \left( \frac{1 - \sqrt{5}}{2} \right)^n \right]
Solution. Let \alpha = \frac{1 + \sqrt{5}}{2}, ~~ \beta = \frac{1 - \sqrt{5}}{2}.
We will prove by induction the following statement F_n = \frac{1}{\sqrt{5}} ( \alpha^n - \beta^n )
For n=0, we have \frac{1}{\sqrt{5}} ( \alpha^0 - \beta^0 ) = 0 = F_0
Thus, the statement is correct for the case n=0.
For n=1, we have \frac{1}{\sqrt{5}} ( \alpha^1 - \beta^1 ) = \frac{1}{\sqrt{5}} \sqrt{5} = 1 = F_1
So the statement is also correct for the case n=1.
Suppose that the statement is correct for all the cases 0 \leq n \leq k where k \geq 1. We will prove that the statement is also correct for the case n=k+1, i.e. we will prove that F_{k+1} = \frac{1}{\sqrt{5}} ( \alpha^{k+1} - \beta^{k+1} )
Indeed, since 0 \leq k-1 \leq k, by the induction assumption, the statement is correct for the case n=k-1, thus, F_{k-1} = \frac{1}{\sqrt{5}} ( \alpha^{k-1} - \beta^{k-1} )
Also by the induction assumption, the statement is correct for the case n=k, so F_{k} = \frac{1}{\sqrt{5}} ( \alpha^{k} - \beta^{k} )
It follows that F_{k+1} = F_{k-1} + F_k = \frac{1}{\sqrt{5}} [ (\alpha^{k-1} + \alpha^{k}) - (\beta^{k-1} + \beta^{k})] = \frac{1}{\sqrt{5}} [ \alpha^{k-1} (1 + \alpha) - \beta^{k-1} ( 1 + \beta)]
Observe that \alpha and \beta are the two roots of the equation 1+x=x^2, so 1+\alpha=\alpha^2 and 1+\beta=\beta^2. It follows that F_{k+1} = \frac{1}{\sqrt{5}} ( \alpha^{k-1} \alpha^2 - \beta^{k-1} \beta^2 ) = \frac{1}{\sqrt{5}} ( \alpha^{k+1} - \beta^{k+1} )
So we have proved that the statement is correct for the case n=k+1.
By the mathematical induction principle, the statement must be correct for any natural number n. \blacksquare
In Problem 6, in order to prove that P(k+1) is correct, we have to use two assumptions that P(k-1) is correct and P(k) is correct. Because of that, in the initial step, we have to verify that P(0) is correct and P(1) is correct. It then follows from the induction step that:
- because P(0), P(1) are correct, we conclude that P(2) is correct
- because P(0), P(1), P(2) are correct, we conclude that P(3) is correct
- because P(0), P(1), P(2), P(3) are correct, we conclude that P(4) is correct
- v.v...
Proving 1 > 2
We will use induction to prove that 1 > 2. Consider the following proof carefully to see if you can point out the wrong step in this proof.
Let us construct the following sequence: a_0 = 1, a_1 = 1, a_{n+1} = a_{n-1} + a_n + 11.
We will prove by induction the following statement
For any n, we have a_n > 4 n - 2
For n=0, we have a_0 = 1 > 4 \times 0 - 2 = -2
So the statement is correct for the case n=0.
Assume that the statement is correct for any 0 \leq n \leq k. We will prove that the statement is also correct for the case n=k+1, that is, we will prove a_{k+1} > 4(k+1) - 2 = 4k + 2
Indeed, by the induction assumption, the statement is correct for the case n=k-1, so
a_{k-1} > 4(k-1) - 2 = 4k - 6
Also by the induction assumption, the statement is correct for the case n=k, so
a_{k} > 4k - 2
It follows that
a_{k+1} = a_{k-1} + a_k + 11 > (4k - 6) + (4k - 2) + 11 = 8k + 3 > 4k + 2
So we have proved that the statement is correct for the case n=k+1.
By the mathematical induction principle, the statement must be correct for any natural number n.
We have just proved the following inequality
a_n > 4 n - 2
With n=1, the inequality is
1 > 2
So what have we done wrong here?!
See you again in the next post when we solve some more problems by mathematical induction.
Homework
1. Generalize the following equality 1 \times 2 \times 3 + 2 \times 3 \times 4 + \dots + n (n+1)(n+2) = \frac{1}{4} n(n+1)(n+2)(n+3).
2. Prove that 25 ~\mid~ 6^n - 5n - 1
Try to generalize this.
3. With the Fibonacci sequence
F_0 = 0, F_1 = 1, F_2 = 1, F_3 = 2, F_4 = 3, F_5 = 5, F_6 = 8, \dots
determine all values of n such that F_n > 3n.